Difference between revisions of "2002 IMO Shortlist Problems/A2"

(discussion link)
m
 
Line 1: Line 1:
 
== Problem ==
 
== Problem ==
  
Let <math> \displaystyle a_1, a_b, \ldots </math> be an infinite sequence of real numbers, for which there exists a real number <math>\displaystyle c </math> with <math> \displaystyle 0 \le a_i \le c </math> for all <math> \displaystyle i </math>, such that
+
Let <math>a_1, a_b, \ldots </math> be an infinite sequence of real numbers, for which there exists a real number <math>c </math> with <math>0 \le a_i \le c </math> for all <math>i </math>, such that
  
 
<center>
 
<center>
Line 9: Line 9:
 
</center>
 
</center>
  
Prove that <math> \displaystyle c \ge 1 </math>.
+
Prove that <math>c \ge 1 </math>.
  
 
== Solutions ==
 
== Solutions ==
Line 15: Line 15:
 
=== Solution 1 ===
 
=== Solution 1 ===
  
For some fixed value of <math> \displaystyle n </math>, let <math> \displaystyle \sigma </math> be the [[permutation]] of the first <math> \displaystyle n </math> natural numbers such that <math> a_{\sigma(1)} , \ldots a_{\sigma(n)} </math> is an increasing sequence.  Then we have
+
For some fixed value of <math>n </math>, let <math>\sigma </math> be the [[permutation]] of the first <math>n </math> natural numbers such that <math> a_{\sigma(1)} , \ldots a_{\sigma(n)} </math> is an increasing sequence.  Then we have
  
 
<center>
 
<center>
 
<math>
 
<math>
 
\begin{matrix}
 
\begin{matrix}
a_{\sigma(n)} - a_{\sigma(1)} &= \displaystyle \sum_{i=1}^{n-1} \left| a_{\sigma(i+1)} - a_{\sigma(i)} \right| \qquad \quad \\
+
a_{\sigma(n)} - a_{\sigma(1)} &=\sum_{i=1}^{n-1} \left| a_{\sigma(i+1)} - a_{\sigma(i)} \right| \qquad \quad \\
&\ge \displaystyle \sum_{i=1}^{n-1} \frac{1}{\sigma(i+1) + \sigma(i)} \qquad (*)
+
&\ge\sum_{i=1}^{n-1} \frac{1}{\sigma(i+1) + \sigma(i)} \qquad (*)
 
\end{matrix}
 
\end{matrix}
 
</math>
 
</math>
Line 31: Line 31:
 
<math>
 
<math>
 
\begin{matrix}
 
\begin{matrix}
\displaystyle \left( \sum_{i=1}^{n-1} \frac{1}{\sigma(i+1) + \sigma(i)} \right) \left( \sum_{i=1}^{n-1} \sigma(i+1) + \sigma(i) \right) & \ge & (n-1)^2 \qquad \qquad \qquad \\
+
\left( \sum_{i=1}^{n-1} \frac{1}{\sigma(i+1) + \sigma(i)} \right) \left( \sum_{i=1}^{n-1} \sigma(i+1) + \sigma(i) \right) & \ge & (n-1)^2 \qquad \qquad \qquad \\
\displaystyle \qquad \qquad \qquad \qquad \qquad \sum_{i=1}^{n-1} \frac{1}{\sigma(i+1) + \sigma(i)} & \ge & \displaystyle \frac{(n-1)^2}{\displaystyle 2 \sum_{i=1}^{n-1} (\sigma_{i}) - \sigma(1) - \sigma(n)} \\
+
\qquad \qquad \qquad \qquad \qquad \sum_{i=1}^{n-1} \frac{1}{\sigma(i+1) + \sigma(i)} & \ge &\frac{(n-1)^2}{2 \sum_{i=1}^{n-1} (\sigma_{i}) - \sigma(1) - \sigma(n)} \\
& \ge & \displaystyle \frac{(n-1)^2}{n(n+1) -3} \qquad \qquad \\
+
& \ge &\frac{(n-1)^2}{n(n+1) -3} \qquad \qquad \\
& \ge & \displaystyle \frac{n-1}{n+3} . \qquad \qquad \qquad \quad
+
& \ge &\frac{n-1}{n+3} . \qquad \qquad \qquad \quad
 
\end{matrix}
 
\end{matrix}
 
</math>
 
</math>
 
</center>
 
</center>
  
Thus for all <math>\displaystyle n </math>, we must have
+
Thus for all <math>n </math>, we must have
  
 
<center>
 
<center>
Line 47: Line 47:
 
</center>
 
</center>
  
and therefore <math>\displaystyle c </math> must be at least 1, Q.E.D.
+
and therefore <math>c </math> must be at least 1, Q.E.D.
  
 
=== Solution 2 ===
 
=== Solution 2 ===
  
We proceed to <math> \displaystyle (*) </math> as in Solution 1.  We now note that by the [[RMS-AM-GM-HM | AM-HM Inequality]],
+
We proceed to <math>(*) </math> as in Solution 1.  We now note that by the [[RMS-AM-GM-HM | AM-HM Inequality]],
  
 
<center>
 
<center>
 
<math>
 
<math>
 
\begin{matrix}
 
\begin{matrix}
\displaystyle \sum_{i=1}^{n-1} \frac{1}{\sigma(i+1) + \sigma(i)} & \ge & \displaystyle \frac{(n-1)^2}{\displaystyle \sum_{i=1}^{n-1} \left[ \sigma(i+1) + \sigma(i) \right] } \qquad \qquad \qquad \\
+
\sum_{i=1}^{n-1} \frac{1}{\sigma(i+1) + \sigma(i)} & \ge &\frac{(n-1)^2}{\sum_{i=1}^{n-1} \left[ \sigma(i+1) + \sigma(i) \right] } \qquad \qquad \qquad \\
& > & \displaystyle \frac{(n-1)^2}{\displaystyle \sum_{i=1}^{n-1} \left[ \sigma(i+1) + \sigma(i) \right] + \sigma(n) + \sigma(1) } \\
+
& > &\frac{(n-1)^2}{\sum_{i=1}^{n-1} \left[ \sigma(i+1) + \sigma(i) \right] + \sigma(n) + \sigma(1) } \\
& = & \displaystyle \frac{(n-1)^2}{n(n+1)} . \qquad \qquad \qquad \qquad \qquad \quad
+
& = &\frac{(n-1)^2}{n(n+1)} . \qquad \qquad \qquad \qquad \qquad \quad
 
\end{matrix}
 
\end{matrix}
 
</math>
 
</math>
 
</center>
 
</center>
  
Thus for any <math> \displaystyle n </math>, we have two <math> \displaystyle a_i </math> that differ by more than <math>  \frac{(n-1)^2}{n(n+1)} </math>.  But this becomes arbitrarily close to 1 as <math> \displaystyle n </math> becomes arbitrarily large.  Hence if we had <math> \displaystyle c < 1 </math>, then we could obtain a contradiction, so we conclude that <math> \displaystyle c \ge 1 </math>, Q.E.D.
+
Thus for any <math>n </math>, we have two <math>a_i </math> that differ by more than <math>  \frac{(n-1)^2}{n(n+1)} </math>.  But this becomes arbitrarily close to 1 as <math>n </math> becomes arbitrarily large.  Hence if we had <math>c < 1 </math>, then we could obtain a contradiction, so we conclude that <math>c \ge 1 </math>, Q.E.D.
  
 
{{alternate solutions}}
 
{{alternate solutions}}
Line 69: Line 69:
 
== Notes ==
 
== Notes ==
  
The chief difficulty of this problem seems to be obtaining <math> \displaystyle (*) </math>; once this result has been obtained, there are many ways to conclude.
+
The chief difficulty of this problem seems to be obtaining <math>(*) </math>; once this result has been obtained, there are many ways to conclude.
  
 
== Resources ==
 
== Resources ==
Line 78: Line 78:
  
 
[[Category:Olympiad Algebra Problems]]
 
[[Category:Olympiad Algebra Problems]]
 +
[[Category:Olympiad Inequality Problems]]

Latest revision as of 17:58, 20 August 2008

Problem

Let $a_1, a_b, \ldots$ be an infinite sequence of real numbers, for which there exists a real number $c$ with $0 \le a_i \le c$ for all $i$, such that

$|a_i - a_j | \ge \frac{1}{i+j} \mbox{ for all }i,\; j \mbox{ with } i \neq j.$

Prove that $c \ge 1$.

Solutions

Solution 1

For some fixed value of $n$, let $\sigma$ be the permutation of the first $n$ natural numbers such that $a_{\sigma(1)} , \ldots a_{\sigma(n)}$ is an increasing sequence. Then we have

$\begin{matrix} a_{\sigma(n)} - a_{\sigma(1)} &=\sum_{i=1}^{n-1} \left| a_{\sigma(i+1)} - a_{\sigma(i)} \right| \qquad \quad \\ &\ge\sum_{i=1}^{n-1} \frac{1}{\sigma(i+1) + \sigma(i)} \qquad (*) \end{matrix}$

Now, by the Cauchy-Schwarz Inequality, we have

$\begin{matrix} \left( \sum_{i=1}^{n-1} \frac{1}{\sigma(i+1) + \sigma(i)} \right) \left( \sum_{i=1}^{n-1} \sigma(i+1) + \sigma(i) \right) & \ge & (n-1)^2 \qquad \qquad \qquad \\ \qquad \qquad \qquad \qquad \qquad \sum_{i=1}^{n-1} \frac{1}{\sigma(i+1) + \sigma(i)} & \ge &\frac{(n-1)^2}{2 \sum_{i=1}^{n-1} (\sigma_{i}) - \sigma(1) - \sigma(n)} \\ & \ge &\frac{(n-1)^2}{n(n+1) -3} \qquad \qquad \\ & \ge &\frac{n-1}{n+3} . \qquad \qquad \qquad \quad \end{matrix}$

Thus for all $n$, we must have

$c \ge \frac{n-1}{n+3} = 1 - \frac{4}{n+3},$

and therefore $c$ must be at least 1, Q.E.D.

Solution 2

We proceed to $(*)$ as in Solution 1. We now note that by the AM-HM Inequality,

$\begin{matrix} \sum_{i=1}^{n-1} \frac{1}{\sigma(i+1) + \sigma(i)} & \ge &\frac{(n-1)^2}{\sum_{i=1}^{n-1} \left[ \sigma(i+1) + \sigma(i) \right] } \qquad \qquad \qquad \\ & > &\frac{(n-1)^2}{\sum_{i=1}^{n-1} \left[ \sigma(i+1) + \sigma(i) \right] + \sigma(n) + \sigma(1) } \\ & = &\frac{(n-1)^2}{n(n+1)} . \qquad \qquad \qquad \qquad \qquad \quad \end{matrix}$

Thus for any $n$, we have two $a_i$ that differ by more than $\frac{(n-1)^2}{n(n+1)}$. But this becomes arbitrarily close to 1 as $n$ becomes arbitrarily large. Hence if we had $c < 1$, then we could obtain a contradiction, so we conclude that $c \ge 1$, Q.E.D.

Alternate solutions are always welcome. If you have a different, elegant solution to this problem, please add it to this page.

Notes

The chief difficulty of this problem seems to be obtaining $(*)$; once this result has been obtained, there are many ways to conclude.

Resources